Integrating Partial Derivatives

Click For Summary
To find the general function f(x,y) that satisfies the given partial differential equations, integrate each equation separately. The first integration yields x^4 - 2x^2y^2 + sin(x) + c(y), while the second gives -2y^2x^2 + y^4 + c(x). To ensure consistency, differentiate the first solution with respect to y and compare it with the second equation to determine the form of c(y). Both solutions must align, indicating that the system of equations has a valid solution. Systematic comparison of derivatives is essential for identifying the correct function that satisfies both equations.
rudders93
Messages
46
Reaction score
0

Homework Statement


Find the general function f(x,y) that satisifes the following first-order partial differential equations

\frac{df}{dx}=4x^3 - 4xy^2 + cos(x)
\frac{df}{dy}=-4yx^2 + 4y^3

The Attempt at a Solution



I integrated both to get:

x^4 - 2x^2y^2 + sin(x) + c(y)

and

-2y^2x^2 + y^4 + c(x)

I'm not to sure what it means by the most general function that satisifes both. Like I'm assuming that means it's just some function that I can sub into both partial derivatives and get the same result? If so, what is a systematic way of doing so?

Thanks!
 
Last edited:
Physics news on Phys.org
Like I'm assuming that means it's just some function that I can sub into both partial derivatives and get the same result?

I don't know what you mean by "get the same result", but it's some function that satisfies both partial differential equations.

Let's focus on your first solution: x^4 - 2x^2y^2 + sin(x) + c(y)

We know that the second differential equation imposes some extra conditions on c(y), so try differentiating this solution with respect to y and comparing with the second differential equation to see what c(y) must be.

You might wonder why I chose the first solution instead of the second. That's because the two give the same answer. If they didn't, the system of equations would have no solution.
 
Question: A clock's minute hand has length 4 and its hour hand has length 3. What is the distance between the tips at the moment when it is increasing most rapidly?(Putnam Exam Question) Answer: Making assumption that both the hands moves at constant angular velocities, the answer is ## \sqrt{7} .## But don't you think this assumption is somewhat doubtful and wrong?

Similar threads

  • · Replies 6 ·
Replies
6
Views
2K
  • · Replies 4 ·
Replies
4
Views
1K
  • · Replies 5 ·
Replies
5
Views
1K
Replies
2
Views
1K
Replies
3
Views
2K
  • · Replies 6 ·
Replies
6
Views
2K
  • · Replies 2 ·
Replies
2
Views
2K
  • · Replies 5 ·
Replies
5
Views
1K
  • · Replies 3 ·
Replies
3
Views
2K
  • · Replies 2 ·
Replies
2
Views
1K